PT3.S4.Q22 - The United States Food and Drug Administration (FDA) regulates...

MISC_K79MISC_K79 Core Member
edited August 2023 in Logical Reasoning 110 karma

Can someone help with analyzing the stimulus?
I am having difficult time identifying the conclusion.
Admin Note: Edited title. Please use the format: "PT#.S#.Q# - brief description of question”

Comments

  • AlexLSAT.AlexLSAT. Alum Member
    797 karma

    We know this:
    - FDA regulates new therapeutic agents into marketplace > critical roll in improving health care
    - academic/government research communities discover and test these agents while FDA places them in the marketplace
    - they can only help patients after transition into marketplace.

    We need an AC that is supported by these three points above

    B is correct because of the last point. the contrapositive of the statement is /marketplace > /help patients which is basically word for word what B states.

Sign In or Register to comment.